mir.pe (일반/어두운 화면)
최근 수정 시각 : 2024-04-14 09:23:37

Lp 공간

르베그 공간에서 넘어옴
해석학· 미적분학
Analysis · Calculus
{{{#!wiki style="margin: 0 -10px -5px; min-height: calc(1.5em + 5px)"
{{{#!folding [ 펼치기 · 접기 ]
{{{#!wiki style="margin: -5px -1px -11px"
<colbgcolor=#26455A>실수와 복소수 실수( 실직선 · 아르키메데스 성질) · 복소수( 복소평면 · 극형식 · 편각) · 근방 · 유계 · 콤팩트성 · 완비성
함수 함수 · 조각적 정의 · 항등함수 · 역함수 · 멱함수 · 다변수함수( 동차함수 · 음함수) · 다가 함수 · 함수의 그래프 · 좌표계 · 닮은꼴 함수 · 극값 · 볼록/오목 · 증감표
초등함수( 대수함수 · 초월함수 · 로그함수 · 지수함수 · 삼각함수) · 특수함수 · 범함수( 변분법 · 오일러 방정식) · 병리적 함수
극한·연속 함수의 극한 · 수열의 극한 · 연속함수 · ε-δ 논법 · 수렴( 균등수렴) · 발산 · 부정형 · 점근선 · 무한대 · 무한소 · 0.999…=1
중간값 정리 · 최대·최소 정리 · 부동점 정리 · 스털링 근사 · 선형근사( 어림)
수열· 급수 수열 · 급수( 멱급수 · 테일러 급수( 일람) · 조화급수 · 그란디 급수( 라마누잔합) · 망원급수( 부분분수분해)) · 그물
오일러 수열 · 베르누이 수열 · 월리스 곱
단조 수렴 정리 · 슈톨츠-체사로 정리 · 축소구간정리 · 급수의 수렴 판정 · 리만 재배열 정리 · 바젤 문제 · 파울하버의 공식 · 오일러-매클로린 공식 · 콜라츠 추측미해결
미분 미분 · 도함수( 이계도함수 · 도함수 일람) · 곱미분 · 몫미분 · 연쇄 법칙 · 임계점( 변곡점 · 안장점) · 매끄러움
평균값 정리( 롤의 정리) · 테일러 정리 · 역함수 정리 · 다르부 정리 · 로피탈 정리
립시츠 규칙 · 뉴턴-랩슨 방법 · 유율법
적분 적분 · 정적분( 예제) · 스틸체스 적분 · 부정적분( 부정적분 일람) · 부분적분( LIATE 법칙 · 도표적분법 · 예제) · 치환적분 · 이상적분( 코시 주요값)
미적분의 기본정리 · 적분의 평균값 정리
리시 방법 · 2학년의 꿈
다변수· 벡터 미적분 편도함수 · 미분형식 · · 중적분( 선적분 · 면적분 · 야코비안) · 야코비 공식
라그랑주 승수법 · 오일러 동차함수 정리 · 선적분의 기본정리 · 스토크스 정리( 발산 정리 · 그린 정리 변분법
미분방정식 미분방정식( 풀이) · 라플라스 변환
측도론 측도 · 가측함수 · 곱측도 · 르베그 적분 · 절대 연속 측도 · 라돈-니코딤 도함수
칸토어 집합 · 비탈리 집합
복소해석 코시-리만 방정식 · 로랑 급수 · 유수 · 해석적 연속 · 오일러 공식( 오일러 등식 · 드 무아브르 공식) · 리우빌의 정리 · 바이어슈트라스 분해 정리 · 미타그레플레르 정리
함수해석 공간 위상벡터공간 · 노름공간 · 바나흐 공간 · 힐베르트 공간 · 거리공간 · Lp 공간
작용소 수반 작용소 · 에르미트 작용소 · 정규 작용소 · 유니터리 작용소 · 컴팩트 작용소
대수 C*-대수 · 폰 노이만 대수
정리 한-바나흐 정리 · 스펙트럼 정리 · 베르 범주 정리
이론 디랙 델타 함수( 분포이론)
조화해석 푸리에 해석( 푸리에 변환 · 아다마르 변환)
관련 분야 해석기하학 · 미분기하학 · 해석적 정수론( 1의 거듭제곱근 · 가우스 정수 · 아이젠슈타인 정수 · 소수 정리 · 리만 가설미해결) · 확률론( 확률변수 · 중심극한정리) · 수치해석학 · 카오스 이론 · 분수계 미적분학 · 수리물리학 · 수리경제학( 경제수학) · 공업수학
양-밀스 질량 간극 가설미해결 · 나비에 스토크스 방정식의 해 존재 및 매끄러움미해결
기타 퍼지 논리
}}}}}}}}} ||

1. 개요2. 정의
2.1. Lp 공간2.2. lp 공간
3. 구조
3.1. 벡터공간3.2. 노름공간
3.2.1. 준노름공간3.2.2. 내적공간3.2.3. 균등 볼록 공간
3.3. 거리공간
3.3.1. Lp 수렴
3.4. 완비공간
4. 성질
4.1. Lp 공간 사이의 관계4.2. Lp 공간의 쌍대공간4.3. Lp 공간의 부등식4.4. 작용소의 보간
5. 적용


[math(L^p)]-space, Lebesgue space

1. 개요

[math(L^p)]공간(르베그 공간)은 측도공간에서 절댓값의 [math(p)]제곱이 르베그 적분 가능한 함수의 공간이다. 이때, 거의 어디에서나 같은 함수들은 동일한 함수로 본다. [math(p\ge1)]일 때 [math(L^p)] 공간은 완비 노름공간, 즉 바나흐 공간이다. 특히 [math(p=2)]일 때에는 완비 내적공간, 즉 힐베르트 공간이다.

2. 정의

2.1. Lp 공간

측도공간 [math( (X,\ \mathcal{M},\ \mu) )]와 실수 [math(p\in(0, \infty])]가 주어졌을 때, 보렐 가측함수 [math(f : X \rightarrow \mathbb{K}\ (\mathbb{K\in\{R,\ C\}}))]에 대하여 범함수 [math(\|\cdot\|_{L^p})](또는 [math(\|\cdot\|_p)]로 표기)를
[math(\|f\|_{L^p}=\begin{cases}\displaystyle\left(\int_X|f|^p d\mu\right)^{1/p}&\text{if }p\in[1,\infty)\\
\text{ess}\sup_{x\in X}|f(x)|&\text{if }p=\infty
\end{cases})]
라 하자. 여기서 [math(\text{ess}\sup_{x\in X}|f(x)|)]는 [math(|f|)]의 본질적 상한으로, [math(\inf \{ M : \mu(|f|>M)=0\})]를 뜻한다. 함수공간
[math(\mathcal{L}^p(X,\ \mathcal{M} ,\ \mu):= \left\{ f : \|f\|_{L^p} <\infty \right\})]
의 동치관계 [math(f\sim_{\text{a.e.}} g\Longleftrightarrow f=g\text{ a.e. })]에 의한 상공간
[math(L^p(X,\mathcal{M},\mu):=\mathcal{L}^p/\sim_{\text{a.e.}})]
(또는 [math(L^p, L^p(X), L^p(\mu))] 등으로 표기)를 [math(L^p)] 공간 또는 르베그-[math(p)] 공간이라고 한다.

2.2. lp 공간

셈 측도공간 [math((A,\mathcal{P}(A),\mu))]와 실수 [math(p\in(0, \infty])]가 주어졌을 때, [math(L^p(A,\mathcal{P}(A),\mu))] 공간을 [math(l^p(A))]로 나타낸다. 특히, [math(A=\mathbb{N})]일 때 [math(l^p(\mathbb{N}))]을 [math(l^p)]로 나타낸다. [math(\mathbb{N})]을 정의역으로 하는 함수는 수열이므로 수열 [math(a_n:\mathbb{N}\to\mathbb{K})]에 대한 범함수 [math(\|\cdot\|_{l^p}=\|\cdot\|_{L^p})]는 다음과 같이 정의한다.
[math(\|a_n\|_{l^p}=\begin{cases}\displaystyle\left(\sum_{n=1}^\infty|a_n|^p \right)^{1/p}&\text{if }p\in[1,\infty)\\
\sup_{n\in\mathbb{N}}|a_n|&\text{if }p=\infty
\end{cases})]

3. 구조

3.1. 벡터공간

[math(p\in(0,\infty))]일 때, [math(f\in \mathcal{L}^p)]와 [math(a\in\mathbb{K})]에 대하여
[math(\displaystyle\int|af|^p=|a|^p\int |f|^p<\infty)]
이므로 [math(af\in \mathcal{L}^p)]이다. 또한 [math(f,g\in \mathcal{L}^p)]에 대하여
[math(\begin{aligned}
|f(x)+g(x)|^p&\le(2\max\{|f(x)|,|g(x)|\})^p\\
&=2^p\max(|f(x)|^p,|g(x)|^p)\\
&\le 2^p(|f(x)|^p+|g(x)|^p)
\end{aligned})]
이므로 [math(f+g\in \mathcal{L}^p)]이다. 이는 거의 모든 곳에서 같은 함수족 사이에서 동일하게 성립하므로 [math(L^p)] 공간은 벡터공간이다.

[math(p=\infty)]일 때, [math(f\in \mathcal{L}^\infty)]와 [math(a\in\mathbb{K},M>0 )]에 대하여 [math(\{x:|f(x)|>M\}=\{x:|af(x)|>|a|M\})]이므로
[math(\mu(\{x:|f(x)|>M\})=0\Longleftrightarrow\mu(\{x:|af(x)|>|a|M\})=0)]
이다. 따라서 [math(\|af\|_\infty=|a|\|f\|_\infty<\infty)]로, [math(af\in \mathcal{L}^\infty)]이다. 또한 [math(f,g\in \mathcal{L}^\infty)]에 대하여 거의 모든 [math(x)]에서
[math(|f(x)+g(x)|\le|f(x)|+|g(x)|\le\|f\|_\infty+\|g\|_\infty)]
이므로 [math(\|f+g\|_\infty<\infty)]이다. 따라서 [math(f+g\in \mathcal{L}^\infty)]이다. [math(p\in(1,\infty))]인 경우와 마찬가지로 위 성질은 거의 모든 곳에서 같은 함수족 사이에서 동일하게 성립하므로 [math(L^\infty)]는 벡터공간이다.

3.2. 노름공간

측도공간 [math((X, \mathcal{M},\mu))]의 가측함수에 대하여 다음 부등식이 성립한다. [math(p>1)]일 때, [math(\mathcal{L}^p)] 공간 위에서 정의된 범함수 [math(\|\cdot\|_{L^p})]는 다음 성질을 만족시켜 [math(\mathcal{L}^p)] 위의 반노름을 이룬다.
[math(L^p)]가 벡터공간임을 증명하는 과정에서 양의 동차성을 보였으며, 삼각 부등식은 민코프스키 부등식에 의해 성립한다. 범함수 [math(\|\cdot\|_{L^p})]가 노름을 이루기 위해서는 양의 정부호성을 추가로 만족시켜야 한다.
[math(f\in \mathcal{L}^p)]에 대하여 [math(\|f\|_{L^p}=0)]이면 거의 모든 [math(x)]에서 [math(f=g)]인 [math(g\in\mathcal{L}^p)]에 대하여 [math(\int|g|=\int|f|=0)]이므로 [math(\|g\|_{L^p}=0)]이다. 즉, [math(\|\cdot\|_{L^p})]는 양의 정부호성을 만족시키지 않아 노름이 아니다. 여기서 반노름공간 [math(\mathcal{L^p})]에 동치관계 [math(\sim_{\text{a.e.}})]에 대한 상공간을 취하면 동치류 [math([f]\in\mathcal{L^p}/\sim_{\text{a.e.}})]에 대하여 [math(f_1, f_2\in[f])]이면 [math(\int|f_1|^p=\int|f_2|^p)]이므로 [math(\|\cdot\|_{L^p})]는 [math(\mathcal{L^p}/\sim_{\text{a.e.}})] 위에서 반노름을 이룬다. 특히, [math(\int|f|^p=0)]이면 [math(f\in[0])]이므로 반노름 [math(\|\cdot\|_{L^p})]는 양의 정부호성을 만족시켜 [math(\mathcal{L^p}/\sim_{\text{a.e.}})] 위의 노름이다. 노름공간 [math(\mathcal{L^p}/\sim_{\text{a.e.}})]를 [math(L^p(X,\mathcal{M},\mu))](또는 [math(L^p(X), L^p(\mu))] 등으로 표기)로 나타낸다. [math(L^p)]의 원소는 엄밀히 함수가 아닌 함수의 동치류이지만, [math(L^p)]공간을 다룰 때 동치류 [math([f])]를 하나의 함수 [math(f)]로 취급한다.

[math(f,g\in\mathcal{L^p})]에 대하여 [math(\|f-g\|_{L^p}=0)]일 필요충분조건은 [math(f\sim_{\text{a.e.}}g)]이므로 [math(L^p)] 공간은 [math(\mathcal{L}^p)]의 부분공간 [math(\mathcal{N}=\{f:\|f\|_{L^p}=0\})]에 의한 상공간과 같다.

3.2.1. 준노름공간

[math(p\in(0, 1))]일 때, [math(\|\cdot\|_{L^p})]에 대하여 반대 민코프스키 부등식[math(\|f+g\|_{L^p}\ge\|f\|_{L^p}+\|g\|_{L^p})]가 성립한다. 따라서 [math(\|\cdot\|_{L^p})]는 반노름을 이루지 않는다. 그러나 [math(\mathcal{L}^p)] 공간이 벡터공간임을 보이는 과정에서 보인 바와 같이 [math(\|\cdot\|_{L^p})]는 양의 동차성을 만족시키며, 임의의 [math(f, g\in\mathcal{L}^p)]에 대하여 [math(\|f+g\|_{L^p}\le K(\|f\|_{L^p}+\|g\|_{L^p}))]를 만족시키는 양수 [math(K)]가 존재하므로 준반노름(quasi semi norm)이다. [math(p\in[1,\infty])]일 때와 마찬가지로 [math(\|\cdot\|_{L^p})]가 양의 정칙성을 만족시켜 준노름을 이룰 수 있도록 준반노름공간 [math(\mathcal{L^p})]에 동치관계 [math(\sim_{\text{a.e.}})]에 의한 상공간을 취하여 준노름공간 [math(L^p)]를 얻는다.

3.2.2. 내적공간

[math(L^2)] 노름은 평행사변형 법칙
[math(\|f+g\|_{L^2}^2+\|f-g\|_{L^2}^2=2\left(\|f\|_{L^2}^2+\|g\|_{L^2}^2\right))]
을 만족시키므로[1] [math(L^2)] 공간은 내적공간이다. [math(L^2)] 공간의 내적은 [math(f, g\in L^2)]에 대하여
[math(\displaystyle\left<f, g\right>=\int f\overline{g})]
로 정의하며 실제로 [math(\left<f, f\right>=\int f\overline{f}=\int |f|^2=\|f\|_{L^2}^2)]이므로 [math(L^2)] 노름은 내적에서 유도된 노름이다.

[math(p\ne 2)]일 때 [math(L^p)] 공간은 일반적으로 내적공간이 아니다. 실수 위의 르베그 측도가 주어졌을 때, [math(f=1_{(0, 1)}, g=1_{(1, 2)})]라고 하면
[math(\begin{aligned}\displaystyle&\|f+g\|_{L^p}^2+\|f-g\|_{L^p}^2\\
&=\left(\int|f+g|^p\right)^{2/p}+\left(\int|f-g|^p\right)^{2/p}\\
&=2\cdot2^{2/p}
\end{aligned})]
이지만 [math(\|f\|_{L^p}^2=1)]이므로 [math(2(\|f\|_{L^p}^2+\|g\|_{L^p}^2)=4)]로 [math(p\ne2)]일 때 [math(\|f+g\|_{L^2}^2+\|f-g\|_{L^2}^2\ne2\left(\|f\|_{L^2}^2+\|g\|_{L^2}^2\right))]이다.

3.2.3. 균등 볼록 공간

[math(1<p<\infty)]일 때, [math(L^p)] 공간은 균등 볼록 공간이다.

3.3. 거리공간

[math(p\ge1)]일 때, 함수 [math(d:L^p\times L^p \to [0, \infty))]를 [math(d(f,g)=\|f-g\|_{L^p})]로 정의한다. [math(d(f,f)=\|f-f\|_{L^p}=\|0\|_{L^p}=0)]이며, [math(f\ne g)]이면 [math(f-g\ne 0)]이므로 임의의 [math(f, g(f\ne g)\in L^p)]에 대하여 [math(d(f, g)>0)]이다. 또한 [math(f-g=g-f)]이므로 [math(d(f, g)=d(g, f))]이며 민코프스키 부등식에 의해 임의의 [math(f, g, h)]에 대하여
[math(\begin{aligned}d(f, h)&=\|f-h\|_{L^p}\\
&=\|f-g+g-h\|_{L^p}\\
&\le\|f-g\|_{L^p}+\|g-h\|_{L^p}\\
&=d(f, g)+d(g, h)
\end{aligned})]
이므로 [math(d)]는 [math(L^p)] 공간의 거리함수이다.

[math(p\in(0, 1))]일 때 함수 [math(d:L^p\times L^p\to[0,\infty))]를 [math(d(f, g)=\|f-g\|_{L^p}^p)]로 정의한다. [math(p\in[1, \infty))]인 경우와 동일하게 정의할 경우, 반노름 [math(\|\cdot\|_{L^p})]는 역 민코프스키 부등식을 만족시켜 거리를 유도할 수 없다. 삼각부등식을 제외한 거리함수의 공리는 [math(p\ge1)]인 경우와 동일하게 만족한다. [math(d)]가 삼각 부등식을 만족시키는 것을 보이기 위하여 [math(p\in (0, 1))]와 [math(a, b\ge0)] 에대하여 [math((a+b)^p\le a^p +b^p)]를 활용한다.
증명
실수 [math(t\ge0)]에 대하여 함수 [math(f)]를 [math(f(t):=(1+t)^p-1-t^p)]라 하자. 임의의 양수 [math(t)]에 대하여 [math(f^\prime(t)=p(1+t)^{p-1}-pt^{p-1})]이고 [math(f(0)=0)]이므로 모든 [math(t>0)]에 대하여 [math(f(t)<0)]이다. [math(a, b\ne0)]일 때, [math(t=a/b)]라 하면 [math((1+a/b)^p-1-(a/b)^p<0)]이므로 [math((a+b)^p<a^p+b^p)]이다. [math(ab=0)]이면 [math((a+b)^p=a^p+b^p)]이다.
위 부등식에 의해 임의의 [math(f, g, h\in L^p)]에 대하여 [math(|f-g+g-h|^p\le|f-g|^p+|g-h|^p)]이므로 [math(\|f-h\|_{L^p}^p\le\|f-g\|_{L^p}^p+\|g-h\|_{L^p}^p)]이다. 즉, [math(d(f, h)\le d(f, g)+d(g, h))]를 만족시켜 [math(d)]는 [math(L^p)] 공간의 거리함수이다.

따라서 [math(L^p)] 공간은 거리위상을 갖춘 거리공간이다. 특히 [math(p\in[1,\infty])]일 때 [math(L^p)] 노름 [math(\|\cdot\|_{L^p})]은 [math(L^p)] 위의 연속사상이다.

3.3.1. Lp 수렴

[math(L^p)]의 함수열 [math(\{f_n\})]이 [math(L^p)] 노름으로부터 유도된 거리에 대하여 코시열인 경우, 즉 임의의 [math(\epsilon>0)]에 대하여 [math(m, n>N)]이면
[math(\|f_n-f_m\|_{L^p}<\epsilon)]
을 만족시키는 [math(N\in\mathbb{N})]이 존재하는 경우 [math(\{f_n\})]를 [math(L^p)] 코시열이라고 한다. 또한 [math(\{f_n\})]가 [math(L^p)] 노름으로부터 유도된 거리에 대하여 함수 [math(f)]로 수렴하는 경우, 즉 임의의 [math(\epsilon>0)]에 대하여 [math(m>N)]이면
[math(\|f_n-f\|_{L^p}<\epsilon)]
을 만족시키는 [math(N\in\mathbb{N})]이 존재하는 경우 [math(\{f_n\})]은 [math(f)]로 [math(L^p)] 수렴하는 수열이라고 한다.

[math(L^p)] 수렴성은 거의 어디에서나 수렴성을 함의하지 않으나, 측도수렴성을 함의한다.

3.4. 완비공간

임의의 [math(p\in(0,\ \infty])]에 대하여 [math(L^p(X))]는 완비공간이다. 특히, [math(p\in[1,\ \infty])]일 경우 [math(L^p)] 공간은 완비 노름공간이므로 바나흐 공간이다.
증명
[math(p\in(0,\infty))]일 때, [math(\{f_n\})]을 [math(L^p)] 공간의 코시열이라고 하면 임의의 [math(k\in\mathbb{N})]에 대하여 [math(d(f_{n_k}, f_{n_{k+1}})<2^{-k})]를 만족시키는 부분열 [math(\{f_{n_k}\})]가 존재한다. 함수 [math(f:X\to\mathbb{K})]를
[math(\displaystyle f=f_{n_1}+\sum_{k=1}^\infty (f_{n_{k+1}}-f_{n_k}))]
라고 하면 임의의 [math(N\in\mathbb{N})]에 대하여 거리함수의 삼각부등식에 의해
[math(\begin{aligned}\displaystyle d(f_{n_{N+1}},f_{n_1})&\le \sum_{k=1}^N d(f_{n_{k+1}}, f_{n_k})\\
&\le\sum_{k=1}^N \frac{1}{2^k}\le1\end{aligned})]
이다. 따라서 단조 수렴 정리에 의해 [math(|f_{n_1}|+\sum_{k=1}^\infty|f_{n_{k+1}}-f_{n_k}|\in L^p)]이므로 지배 수렴 정리에 의해 [math(f\in L^p)]이다. [math(\lim_{k\to\infty}f_{n_k}=f)]이므로 [math(\{f_n\})]은 [math(f)]로 수렴하는 부분열을 갖는 코시열로, [math(\{f_n\})]은 [math(f)]로 수렴한다. 즉 [math(L^p)] 공간은 바나흐 공간이다.

[math(p=\infty)]일 때, [math(L^\infty)] 공간의 급수 [math({\sum_{n=1}^\infty f_n})]가 [math(L)]로 절대수렴한다고 하자. [math(\sum_{n=1}^\infty |f_n(x)|>L)]이면 어떤 [math(n)]에 대하여 [math(|f_n(x)|>\|f_n\|_\infty)]이다. 즉
[math(\displaystyle\left\{x:\sum_{n=1}^\infty|f_n (x)|> L\right\}\subseteq \bigcup_{n=1}^\infty\left\{x:|f_n(x)|>\|f_n\|_\infty\right\})]
가 성립한다. 따라서
[math(\displaystyle\mu\left(\left\{x:\sum_{n=1}^\infty|f_n (x)|> L\right\}\right)\le \sum_{n=1}^\infty\mu\left(\left\{x:|f_n(x)|>\|f_n\|_\infty\right\}\right))]
이고 [math(\sum_{n=1}^\infty\mu\left(\left\{x:|f_n(x)|>\|f_n\|_\infty\right\}\right)=0)]이므로 [math(\|\sum_{n=1}^\infty |f_n|\|_\infty\le L)]이다. [math(\left|\sum_{n=1}^\infty f_n\right|\le \sum_{n=1}^\infty|f_n|)]이므로 [math(\sum_{n=1}^\infty f_n\in L^\infty)]이다. 즉 [math(\sum_{n=1}^\infty f_n)]가 [math(L^\infty)] 수렴하여 [math(L^\infty)] 공간은 바나흐 공간이다.
특히, [math(L^2)] 공간은 완비 내적 공간인 힐베르트 공간(Hilbert space)이다. 단순함수, 연속함수, 매끄러운 함수(smooth functions)들의 집합 등은 [math(L^p)]의 조밀한 부분집합을 이룬다.

4. 성질

4.1. Lp 공간 사이의 관계

[math(L^p)] 공간 사이에서 일반적으로 성립하는 포함 관계는 없다. 르베그 측도 공간 [math((0,\ \infty))] 위의 함수 [math(f_a(x)=x^{-a}(a>0))]에 대하여 [math(f_a1_{(0,\ 1)}\in L^p)]일 필요충분조건은 [math(p<a^{-1})]이고 [math(f_a1_{(1,\ \infty)}\in L^p)]의 필요충분조건은 [math(p>a^{-1})]이다. 따라서 [math(p<q)]일 때 [math(p<a^{-1}<q)]이면 [math(f_a1_{(0,\ 1)})]는 [math(L^p)]의 원소이지만 [math(L^q)]의 원소가 아니며, [math(f_a1_{(1,\ \infty)})]는 [math(L^q)]의 원소이지만 [math(L^p)]의 원소가 아니다.

[math(L^p)] 공간 사이의 단순한 포함관계는 일반적으로 성립하지 않으나, [math(1 < p_1 < q < p_2\le\infty)]에 대하여
[math(L^{p_1} \cap L^{p_2} \subset L^q \subset L^{p_1} + L^{p_2})]
가 성립하며, 특히 [math(1/q=(1-\theta )/p_1+\theta /p_2)]인 [math(0\le\theta \le1)]에 대하여
[math(\|f\|_{L^q}\le\|f\|_{L^{p_1}}^{1-\theta}\|f\|_{L^{p_2}}^{\theta })]
이다. 두 벡터공간 [math(L^{p_1}\cap L^{p_2})]와 [math(L^{p_1}+L^{p_2})]는 각각 노름 [math(\|f\|=\|f\|_{L^{p_1}}+\|f\|_{L^{p_2}})]와 [math(\|f\|=\inf\{\|g\|_{L^{p_1}}+\|h\|_{L^{p_2}}:f=g+h\})]가 부여된 바나흐 공간이다.
증명
([math(L^q \subset L^{p_1} + L^{p_2})]) [math(f\in L^q)]에 대하여 [math(E=\{x:|f(x)|>1\})]이라 하자. [math(g=f1_E, h=f1_{E^c})]라 하면 [math(f=g+h)]이다. [math(|g|^{p_1}=|f|^{p_1}1_E\le|f|^q1_E)]이므로 [math(g\in L^p_1)]이고 [math(|h|^{p_2}=|f|^{p_2}1_{E^C}\le|f|^q 1_{E^C})]이므로 [math(h\in L^{p_2})]이다. 따라서 [math(L^q\subset L^{p_1}+L^{p_2})]이다.

([math(L^{p_1} \cap L^{p_2} \subset L^q)]) [math(p_2=\infty)]일 때, [math(q-p_1>0)]이므로 [math(|f|^{q-p_1}\le \|f\|_\infty^{q-p_1})]이고 따라서 [math(|f|^q\le \|f\|_\infty^{q-p_1}|f|^{p_1})]가 성립한다. 부등식의 양변을 적분하고 [math(\theta =(q-p_1)/q)]라 하면 다음이 성립한다.
[math(\|f\|_{L^q}\le \|f\|_{L^{p_1}}^{p_1/q}\|f\|_\infty^{(q-p_1)/q}=\|f\|_{L^{p_1}}^{(1-\theta )}\|f\|_\infty^{\theta})]
[math(p_2<\infty)]일 때, [math(q^{-1}=(1-\theta ) p_1^{-1}+\theta p_2^{-1})]이므로 [math(p_1/(1-\theta ) q)]와 [math(p_2/\theta q)]는 횔더 켤레이다. 횔더 부등식에 의해
[math(
\displaystyle\begin{aligned}
&\quad\int |f|^q\\
&=\int|f|^{(1-\theta ) q}|f|^{\theta q}\\
&\le \|\,|f|^{(1-\theta ) q}\,\|_{p_1/(1-\theta ) q}\|\,|f|^{\theta q}\,\|_{p_2/\theta q}\\
&=\left(\int|f|^{p_1}\right)^{(1-\theta ) q/p_1}\left(\int|f|^{p_2}\right)^{\theta q/p_2}\\
&=\|f\|_{L^{p_1}}^{(1-\theta ) q}\|f\|_{L^{p_2}}^{\theta q}
\end{aligned})]
가 성립한다. 위 부등식의 양 변에 [math(q)]제곱근을 취하여 [math(\|f\|_{L^q}\le\|f\|_{L^{p_1}}^{(1-\theta )}\|f\|_{L^{p_2}}^{\theta})]를 얻는다. 즉, [math(f\in L^{p_1}\cap L^{p_2})]이면 [math(f\in L^q)]로, [math(L^{p_1}\cap L^{p_2}\subset L^q)]이다.

(바나흐 공간 [math(L^{p_1}\cap L^{p_2})]) [math(f\in L^{p_1}\cap L^{p_2})]에 대하여 [math(f\in L^{p_1}, f\in L^{p_2})]이고 [math(\|\cdot\|_{L^{p_1}})]과 [math(\|\cdot\|_{L^{p_2}})]는 각각 [math( L^{p_1}, L^{p_2})]의 노름이므로 [math(\|\cdot\|_{L^{p_1}}+\|\cdot\|_{L^{p_2}})]는 [math(f\in L^{p_1}\cap L^{p_2})] 위에서 노름의 공리를 만족시킨다. [math(\{f_n\})]을 [math(L^{p_1}\cap L^{p_2}\subset L^{q})]의 코시열이라 하면 [math(n, m\to\infty)]에 따라 [math(\|f_n-f_m\|_{L^{p_1}}+\|f_n-f_m\|_{L^{p_2}}\to0)]이므로 [math(\{f_n\})]은 [math(L^{p_1}, L^{p_2})]의 코시열이며 [math(L^{p_1}, L^{p_2})]는 바나흐 공간이므로 [math(\{f_n\})]의 [math(L^{p_1})] 극한 [math(g_1)], [math(L^{p_2})] 극한 [math(g_2)]가 존재한다. [math(L^p)] 수렴 함수열은 측도수렴하며, 측도수렴의 극한은 유일하므로 거의 모든 [math(x)]에서 [math(g_1(x)=g_2(x))]이다. 즉, [math(f:=g_1=g_2)]는 [math(L^{p_1}\cap L^{p_2})]에 속하여 [math(L^{p_1}\cap L^{p_2})]는 바나흐 공간이다.

(바나흐 공간 [math(L^{p_1}+ L^{p_2})]) [math(f\in L^{p_1}+L^{p_2})]에 대하여 [math(f=g+h(g\in L^{p_1}, h\in L^{p_2}))]라고 하면 상수 [math(a\in\mathbb{K})]에 대하여 [math(af=ag+ah)]이므로
[math(\begin{aligned}\|af\|&=\inf\{\|ag\|_{L^{p_1}}+a\|ah\|_{L^{p_2}}\}\\
&=|a|\inf\{\|g\|_{L^{p_1}}+\|h\|_{L^{p_2}}\}\\
&=|a|\|f\|\end{aligned})]
이다. 또한 [math(f, g\in L^{p_1}+L^{p_2})]가 [math(f=f_1+f_2, g=g_1+g_2(f_1, g_2\in L^{p_1}, f_2, g_2\in L^{p_2}))]일 때, [math(\|f_i+g_i\|_{p_i}\le\|f\|_{p_i}+\|g\|_{p_i}(i=1, 2))]이므로 [math(\|f+g\|\le\|f\|+\|g\|)]가 성립한다. [math(\|f\|=0)]이면 [math(\|g_n\|_{L^{p_1}}+\|h_n\|_{L^{p_2}}\searrow0, g_n+h_n=f)]를 만족시키는 [math(L^{p_1})]의 함수열 [math(\{g_n\})]과 [math(L^{p_2})]의 함수열 [math(\{h_n\})]이 존재한다. 노름의 연속성에 의해 [math(\{g_n\})]과 [math(\{h_n\})]은 [math(0)]으로 각각 [math(L^{p_1})] 수렴, [math(L^{p_2})] 수렴하므로 [math(f=0)]이다. 따라서 [math(\|\cdot\|)]는 [math(f\in L^{p_1}+L^{p_2})]의 노름이다. [math(\sum_{n=1}^\infty f_n=\sum_{n=1}^\infty(g_n+h_n))]을 [math(f\in L^{p_1}+L^{p_2})]의 절대수렴 급수라 하자. 즉,
[math(\displaystyle\sum_{n=1}^\infty\|f_n\|=\sum_{n=1}^\infty\left(\inf\{\|g_n\|_{L^{p_1}}+\|h_n\|_{L^{p_2}}\}\right))]
가 수렴한다. 각 [math(n\in\mathbb{N})]에 대하여 [math(\|r_n\|_{L^{p_1}}+\|s_n\|_{L^{p_2}}\le\|f_n\|+2^{-n}, f_n=r_n+s_n)]인 [math(r_n\in L^{p_1}, s_n\in L^{p_2})]를 택하면 부등식의 우변의 급수가 [math(\sum_{n=1}^\infty \|f_n\|+1)]로 수렴하므로 [math(\{r_n\}, \{s_n\})]의 급수는 절대수렴 하여 각각 [math(L^{p_1}, L^{p_2})] 수렴한다. [math(g=\sum_{n=1}^\infty r_n\in L^{p_1}, h=\sum_{n=1}^\infty s_n\in L^{p_2})]에 대하여 [math(f=g+h)]라 하면
[math(\displaystyle
\left\|f-\sum_{k=1}^n f_n\right\|=\inf\left\{\left\|g-\sum_{k=1}^n g_n\right\|_{L^{p_1}}+\left\|h-\sum_{k=1}^n h_n\right\|_{L^{p_2}}\right\}\\
\quad\ \,\le \left\|\sum_{m=n+1}^\infty r_m\right\|_{L^{p_1}}+\left\|\sum_{m=n+1}^\infty s_m\right\|_{L^{p_2}})]
이고 [math(n\to\infty)]에 따라 우변이 [math(0)]으로 수렴하므로 [math(\sum_{n=1}^\infty f_n)]은 [math(f)]로 수렴한다. 즉, [math(L^{p_1}+ L^{p_2})]는 바나흐 공간이다.
특별한 조건을 추가한 경우 [math(L^p)] 공간 사이에 포함 관계가 성립되기도 한다. [math(\mu)]가 유한측도인 경우 [math({0<p<q\le\infty})]일 때 [math({L^p\supset L^q})]와 [math(\|f\|_{L^p} \le \|f\|_{L^q} \mu (X)^{\frac{1}{p}-\frac{1}{q}})]가 성립한다. 이에 따라 유한측도에서 [math(L^{\infty})] 노름은 [math(p \rightarrow \infty)]일 때 [math(L^p)] 노름의 극한이 된다.
증명
[math(q=\infty)]일 때, 임의의 [math(p\in(0, \infty))]에 대하여
[math(\displaystyle\|f\|_{L^p}^p=\int|f|^p\le \|f\|_\infty^p\int 1 =\|f\|_\infty^p\mu(x))]
이므로 위 성질이 성립한다.

[math(q<\infty)]인 경우 켤레 지수 [math(q/p)]와 [math(q/(q-p))]에 대하여 횔더 부등식을 적용하여 다음을 얻는다.
[math(\displaystyle\begin{aligned}
\|f\|_{L^p}^p =\int |f|^p\cdot 1 &\le \|\,|f|^p\,\|_{q/p}\|1\|_{q/(q-p)}\\
&=\|f\|_{L^q}^p\mu(X)^{(q-p)/q}
\end{aligned})]
즉, [math(L^q)] 노름이 유한한 함수 [math(f)]는 [math(L^p)]노름 또한 유한하여 [math(L_q(\mu)\subset L_p(\mu))]가 성립한다.
또한 셈 측도공간에서 [math({0<p<q\le\infty})]일 때 [math({l^p\subset l^q })]와 [math({\|a_n\|_{L^p}\ge\|a_n\|_{L^q}})]가 성립한다.
증명
[math(q=\infty)]일 때, 임의의 [math(p\in(0, \infty))]에 대하여
[math(\displaystyle\|a_n\|_{L^p}^p=\sup_{n\in\mathbb{N}}|a_n|^p\le\sum_{n=1}^{\infty}|a_n|^p)]
이므로 [math(\|a_n\|_\infty\le \|a_n\|_{L^p})]이다.

[math(q<\infty)]인 경우 [math(p<q<\infty)]이므로 [math(\theta=(q-p)/q)]에 대하여 성질 [math(\|a_n\|_{L^q}\le\|a_n\|_{L^{p_1}}^{1-\theta}\|a_n\|_{L^{p_2}}^{\theta})]를 적용하면
[math(\displaystyle\|a_n\|_{L^q}\le\|a_n\|_{L^p}^{1-\theta} \|a_n\|_\infty^{\theta}\le \|a_n\|_{L^p})]
가 성립한다. 즉, [math(l^p)] 노름이 유한한 수열 [math(a_n)]은 [math(l^q)]노름 또한 유한하여 [math(l_p\subset l_q)]가 성립한다.

4.2. Lp 공간의 쌍대공간

[math(p\in(1, \infty))]와 [math(p)]의 횔더 켤레 [math(q)]에 대하여 [math(L^p)]의 쌍대공간은 [math(L^q)]이다. 즉, [math(L^p)] 공간의 연속 선형 범함수의 집합 [math((L^p)^*=L(L^p, \mathbb{K}))]와 [math(L^q)] 공간 사이에 전단사 등거리 사상이 존재하여 [math(L^p)] 공간의 범함수는 [math(L^q)] 공간의 함수로 결정되며, 그 역도 성립한다.

[math(g\in L^q)]에 대하여 선형 범함수 [math(\phi_g:L^p\to\mathbb{K})]를
[math(\displaystyle \phi_g(f)=\int fg)]
로 정의하면 [math(\phi_g)]는 유계로, [math(\|\phi_g\|=\|g\|_{L^q})]가 성립한다. 여기서 [math(\phi_g)]의 작용소 노름은 [math(\|\phi_g\|=\sup\left\{\left|\int fg\right|:\|f\|_{L^p}=1\right\})]이다. 또한 [math(L^q)]에의 각 원소에 [math((L^p)^*)]의 원소 [math(\phi_g)]를 대응시키는 사상 [math(g\mapsto\phi_g)]는 단사사상이다. 이는 [math(\mu)]가 반유한 측도인 경우 [math(q=\infty)]일 때에도 성립한다.
증명
[math(|\phi_g(f)|\le\int|fg|)]에서 횔더 부등식에 의해 [math(\|\phi_g\|\le\|f\|_{L^p}\|g\|_{L^q}=\|g\|_{L^q} )]이다. [math(g=0)]이면 자명하므로 [math(g\ne0)]이라 하자. [math(L^p)]의 함수
[math(\displaystyle f=\frac{|g|^{q-1}\overline{\mathrm{sgn}\,g}}{\|g\|_{L^q}^{q-1}})]
에 대하여 [math(|\overline{\mathrm{sgn}\,g}|=1)]이고 [math((q-1)p=q)]이므로
[math(\displaystyle\|f\|_{L^p}^p=\frac{\int|g|^{(q-1)p}}{\|g\|_{L^q}^{(q-1)p}}=\frac{\int|g|^q}{\int|g|^q}=1)]
이다. [math(\overline{\mathrm{sgn}\,g}\cdot g=|g| )]이므로
[math(\displaystyle\|\phi_g\|\ge\int fg=\frac{\int |g|^q}{\|g\|_{L^q}^{q-1}}=\|g\|_{L^q})]
이다. 즉, [math(\|\phi\|=\|g\|_{L^q})]이다.

[math(\mu)]가 반유한 측도이고 [math(q=\infty)]인 경우 임의의 [math(\epsilon>0)]에 대하여 [math(A=\{x:|g(x)|>\|g\|_\infty-\epsilon\})]이라 하면 [math(\mu(A)>0)]이므로 [math(0<\mu(B)<\infty)]인 [math(B\subset A)]가 존재한다. [math(f=\mu(B)^{-1}1_B\text{sgn}\,g)]라 하면 [math(\|f\|_{L^1}=\mu(B)^{-1}\mu(B)=1)]이므로
[math(\displaystyle \|\phi_g\|\ge \int fg =\frac{1}{\mu(B)}\int_B |g| \ge \|g\|_\infty -\epsilon)]
이고, [math(\epsilon)]은 임의의 양수이므로 [math(\|\phi_g\|=\|g\|_\infty)]이다.

[math(g_1, g_2\in L^q)]에 대하여 [math(\phi_{g_1}=\phi_{g_2})]라고 하면 임의의 [math(f\in L^p)]에 대하여 [math(\int fg_1=\int fg_2)]로, [math(\int f(g_1-g_2)=0)]이다. 즉, [math(g_1=g_2)]로 사상 [math(g\mapsto\phi_g)]는 단사이다.
또한 사상 [math(g\mapsto\phi_g)]는 전사 사상이다. 즉, [math(p\in(1, \infty))]와 [math(q)]가 횔더 켤레일 때, 각 [math(\phi\in(L^p)^*)]에 대하여 [math(\phi(f)=\int fg)]를 만족시키는 [math(g\in L^q)]가 존재한다. 이는 [math(\mu)]가 [math(\sigma)]-유한 측도인 경우 [math(p=1)]일 때에도 성립한다.
증명
([math(\mu)]가 유한 측도인 경우) [math(\mu)]가 유한측도이므로 모든 단순함수는 [math(L^p)]의 원소이다. [math(\phi\in(L^p)^*)]와 가측집합 [math(E)]에 대하여 [math(\nu(E)=\phi(1_E))]라고 하자. 임의의 서로소 집합렬 [math(\{E_n\})]에 대하여 [math(E=\bigcup_{n=1}^\infty E_n)]이라 하면 [math(\mu(E)=\mu(\bigcup_{n=1}^\infty1_{E_n})=\sum_{n=1}^\infty\mu(E_n)<\infty)]이므로 [math(m, n\to\infty(m>n))]일 때
[math(\displaystyle\left\|\sum_{k=1}^m1_{E_k} -\sum_{k=1}^n1_{E_k}\right\|_{L^p}=\left\|\sum_{k=n+1}^m1_{E_k}\right\|_{L^p}
=\mu\left(\bigcup_{k=n+1}^m E_k\right)^{1/p}\to0)]
이다. 즉, [math(\{\sum_{k=1}^n E_k\})]는 [math(L^p)] 코시열이므로 [math(1_E=\sum_{n=1}^\infty 1_{E_n}\in L^p)]이다. [math(\phi)]는 선형성과 연속성을 가지므로
[math(\displaystyle\nu(E)=\phi(1_E)=\phi\left(\sum_{n=1}^\infty1_{E_n}\right)=\sum_{n=1}^\infty \phi(1_{E_n})=\sum_{n=1}^\infty\nu(E_n))]
이다. 즉, [math(\nu)]는 임의의 서로소 집합렬 [math(\{E_n\})]에 대하여 [math(\nu(\bigcup_{n=1}^\infty E_n)=\sum_{n=1}^\infty \nu(E_n))]을 만족시켜 복소 측도이다. 또한 [math(\mu(E)=0)]이면 [math(1_E=0\in L^p)]이므로 [math(\nu(E)=0)]이다. 즉, [math(\nu)]는 [math(\mu)]에 대하여 절대 연속 측도이다. 따라서 라돈-니코딤 정리에 의해
[math(\displaystyle\phi(1_E)=\nu(E)=\int_E g \,d\mu)]
를 만족시키는 [math(g\in L^1(\mu))]가 존재한다. 따라서 모든 단순함수 [math(f)]에 대하여 [math(\phi(f)=\int fg\,d\mu)]이다. [math(L^p)] 공간에서 단순함수 공간은 조밀한 집합을 이루므로 임의의 [math(f\in L^p)]에 대하여 [math(\phi(f)=\int fg)]를 만족시킨다. 함수 [math(g)]가 [math(L^q)]의 원소임을 보인다. [math(|g_n|\le |g|)]이고 [math(g)]로 수렴하는 함수열 [math(\{g_n\})]에 대하여
[math(\displaystyle f_n=\frac{|g_n|^{q-1}\overline{\mathrm{sgn}\,g}}{\|g_n\|_{L^q}^{q-1}})]
라고 하면 [math(\|f_n\|_{L^p}=1)]이므로 [math(|\phi(f_n)|\le\|\phi\|)]이다. 파투 보조정리에 의해
[math(\displaystyle \|g\|_{L^q}\le \lim\inf\|g_n\|_{L^q}=\lim\inf\int|f_ng_n|\\
\le\lim\inf\int|f_ng|=\lim\inf\int f_ng\le\|\phi\|)]
이다. 따라서 [math(g\in L^q)]이다. 또한 횔더 부등식에 의해 [math(|\int fg|\le\int |fg|\le \|f\|_{L^p}\|g\|_{L^q})]이므로 [math(\|\phi\|\|f\|_{L^p}\le\|f\|_{L^p}\|g\|_{L^q})]에서 [math(\|\phi\|\le\|g\|_{L^q})]이다. 즉, [math(\|\phi\|=\|g\|_{L^q})]이다. 따라서 각 [math(\phi\in(L^p)^*)]에 대하여 [math(\phi(f)=\int fg, \|\phi\|=\|g\|_{L^q})]인 [math(g\in L^q)]가 존재하므로 [math(g\mapsto\phi_g)]는 전사 사상이다.

([math(\mu)]가 [math(\sigma)]-유한 측도인 경우) 자연수 [math(n)]에 대하여 집합열 [math(\{E_n\})]이
[math(\displaystyle0<\mu(E_n)<\infty, E_n\subseteq E_{n+1}, X=\sum_{n=1}^\infty E_n)]
을 만족시킨다 하자. [math(E_n)]은 유한 측도 공간이므로 [math(\phi\in (L^p)^*)]와 임의의 [math(f\in L^p(E_n))]에 대하여 [math(\phi(f)=\int f g_n)]을 만족시키는 [math(g_n\in L^q(E_n))]가 [math(\mu\text{-a.e.})] 유일하게 존재하며, 이러한 [math(g_n)]에 대하여 [math(\|g_n\|_{L^q}=\|\phi|_{L^p(E_n)}\|\le \|\phi\|)]이다. 또한 [math(n<m)]일 때 거의 모든 [math(E_n)]에서 [math(g_m=g_n)]이므로 [math(g)]를 [math(E_n)]에서 [math(g=g_n)]으로 정의한다. 단조 수렴 정리에 의해 [math(\|g\|_{L^q}\le\|\phi\|)]이므로 [math(g\in L^q)]이다. [math(f\in L^p)]에 대하여 지배 수렴정리에 의해 [math(f1_{E_n})]는 [math(f)]로 [math(L^p)]수렴하므로
[math(\displaystyle\phi(f)=\lim_{n\to\infty}\phi(f1_{E_n})=\lim_{n\to\infty}\int_{E_n}fg=\int fg)]
이고, 횔더 부등식에 의해 [math(\|\phi\|\le \|g\|_{L^q})]이므로 [math(\|\phi\|=\|g\|_{L^q})]이다. 따라서 [math(g\mapsto\phi_g)]는 전사 사상이다.

([math(\mu)]가 임의의 측도인 경우) [math(p>1)]일 때, [math(E)]가 [math(X)]의 [math(\sigma)]-유한 부분집합이면 모든 [math(f\in L^p(E))]에 대하여 [math(\phi(f)=\int fg_E, \|g_E\|_{L^q} \le \|\phi\|)]인 [math(g_E\in L^q(E))]가 [math(\mu\text{a.e.})]-유일하게 존재한다. 또한 [math(F)]가 [math(F\supset E)]인 [math(\sigma)]-유한 부분집합이면 [math(E)]에서 [math(g_F|_E=g_E\text{ a.e.}, \|g_F\|_{L^q}\ge\|g_E\|_{L^q})]이다.
[math(M=\sup\{\|g_E\|_{L^q}:E\text{ is }\sigma\text{-finite}\})]
라고 하면 임의의 [math(\sigma)]-유한 부분집합 [math(E)]에 대하여 [math(\|g_E\|_{L^q}\le\|\phi\|)]이므로 [math(M\le\|\phi\|)]이다. [math(\|g_{E_n}\|_{L^q})]가 [math(M)]으로 수렴하는, [math(X)]의 [math(\sigma)]-유한 부분집합렬 [math(\{E_n\})]에 대하여 [math(F=\bigcup_{n=1}^\infty E_n)]이라 하면 [math(F)]는 [math(X)]의 [math(\sigma)]-유한 부분집합이고 [math(\|g_F\|_{L^q}=M)]이다. [math(A)]가 [math(F)]를 포함하는, [math(X)]의 [math(\sigma)]-유한 부분집합이면
[math(\displaystyle\int |g_A|^q=\int|g_F|^q=+\int|g_{A\setminus F}|^q \le M^q=\int |g_F|^q)]
이므로 [math(g_{A\setminus F}=0)]으로, [math(g_A=g_F\text{ a.e.})]이다. 각 [math(f\in L^p)]에 대하여 [math(\mathrm{supp}(f))]는 [math(\mu)]-가측이므로 [math(A=F\cup \mathrm{supp}(f))]는 [math(X)]의 [math(\sigma)]-유한 부분집합이고,
[math(\phi(f)=\phi(f1_{A}+f1_{A^c})=\phi(f1_{A})+\phi(f1_{A^c})=\phi(f1_A))]
이므로 [math(\phi(f)=\int fg_A=\int fg_F)]이다. 따라서 [math(g=g_F)]라 하면 횔더 부등식에 의해 [math(\|\phi\|\le\|g\|_{L^q})]이므로 [math(g\mapsto\phi_g)]는 전사 사상이다.
이에 따라 [math(1<p<\infty)]일 때 사상 [math(g\mapsto\phi_g)]는 [math(L^q)]와 [math((L^p)^*)] 사이의 전단사 거리동형이므로 [math((L^p)^{**}=L^p)]이다. 즉, [math(L^p)]공간은 반사적이다.

이 쌍대성은 [math(L^1)]과 [math(L^{\infty})]사이에서는 성립하지 않는다. [math(L^1)]에서 [math((L^{\infty})^*)]로의 사상 [math(g\mapsto\phi_g)]는 등거리 단사 사상 이지만 이 사상은 일반적으로 전사가 아니다. 마찬가지로 [math(L^\infty)]에서 [math((L^1)^*)]로의 사상 [math(g\mapsto\phi_g)]는 일반적으로 전단사가 아니다. [math(L^\infty)] 노름의 정의에서 영측도 조건을 국소적 영측도로 교체하여 단사성을, 이 조건에 분해가능 측도 조건이 추가로 만족되면 전사성을 얻는다.

[math(p\in(0,1))]일 때, [math(\mu)]가 비원자적 측도인 경우 [math((L^p)^*=\{0\})]이다. 이는 노름공간의 비자명 연속 선형 범함수 공간의 존재를 보장하는 한-바나흐 정리가 준노름 공간에서는 성립하지 않음을 보이는 대표적인 반례이다.
증명
비원자적 측도공간 [math((X,\mathcal{M},\mu))]와 실수 [math(p\in (0, 1))]에 대하여 [math(L^p(X,\mu))]는 비자명 볼록 열린 집합을 갖지 않아 국소적 볼록 공간이 아님을 보인다. [math(U)]를 원점의 공집합이 아닌 볼록 열린 집합이라 하자. [math(U)]는 열린집합이므로 [math(0)]을 원소로 갖는 열린 공 [math(B_\delta\subseteq E)]가 존재한다. 임의의 [math(f\in L^p(X))]에 대하여
[math(\displaystyle\frac{\|f\|_{L^p}^p}{n^{1-p}}\ge\delta)]
를 만족시키는 양의 정수 [math(n)]을 택한다. [math(\mu)]는 비원자적 측도이므로 사잇값 성질을 갖고, 따라서 다음을 만족시키는 가측집합 [math(E_1)]이 존재한다.
[math(\displaystyle\int_{E_1}|f|^p\, d\mu =\frac{1}{n}\int_X |f|^p\,d\mu=\frac{\|f\|_{L^p}^p}{n})]
함수 [math(f_1=f\cdot 1_{E_1^c})]에 대하여 위 과정을 반복하면 [math(1\le k\le n)]에 대하여
[math(\displaystyle\int_{E_k}|f|^p\, d\mu=\frac{\|f\|_{L^p}^p}{n})]
을 만족시키는 [math(X)]의 서로소 가측집합으로의 분할 [math(\{E_1,\ldots,E_n\})]을 얻는다. 함수 [math(h_k)]를 [math(h_k:=nf\cdot1_{E_k})]로 정의하면
[math(\displaystyle\int_X|h_k|^p\, d\mu=\int_{E_k}n^p|f|p\,d\mu=\frac{1}{n^{1-p}}\int_X |f|^p\, d\mu\ le \delta)]
이다. 따라서 각 [math(k)]에 대하여 [math(h_k\in B_\delta\subset U)]이고 볼록성에 의하여 [math(f=n^{-1}\sum_{k=1}^n h_k \in U)]이다. [math(f)]는 [math(L^p(X, \mu))]의 임의의 함수이므로 [math(U=L^p(X, \mu))]이다.

다음으로 [math(L^p(X, \mu))]에서 국소적 볼록 [math(T_0)] 공간 [math(Y)]로의 연속 선형 범함수 [math(T)]가 [math(T=0)]임을 보인다. [math(\mathcal{B})]를 [math(Y)]의 원점의 볼록 국소 기저라고 하면 임의의 [math(B\in \mathcal{B})]에 대하여 [math(T^{-1}(B)=L^p(X,\mu))]이므로 임의의 [math(B\in \mathcal{B})]에 대하여 [math(T(L^p(X,\mu))\subset B)]이다. [math(x\in \bigcap_{B\in\mathcal{B}}B)]가 [math(x\ne 0)]이라 가정하면 [math(Y\setminus\{x\})]는 [math(0)]의 열린근방이므로 [math(\bigcap_{B\in\mathcal{B}}B\subset Y\setminus\{x\})]이다. 이는 모순으로, [math(\bigcap_{B\in\mathcal{B}}B=\{0\})]이다. 즉, [math(T(L^p(X,\mu))=\{0\})]이다. [math(\mathbb{K})]는 국소적 볼록 [math(T^1)] 공간이므로 [math(L^p(X, \mu)^*=\{0\})]이다.

4.3. Lp 공간의 부등식

[math(\|f\|_{L^q}\le\|f\|_{L^{p_1}}^{(1-\theta )}\|f\|_{L^{p_2}}^{\theta })]
}}}이다. 이는 사상 [math(1/p\mapsto\|f\|_{L^p})]의 로그 볼록성을 의미하는 것으로 횔더 부등식과 동치이다.
[math(L^p)] 공간 사이의 관계에서 횔더 부등식에서 [math(L^p)] 노름의 로그 볼록성을 유도하는 과정이 증명되었다. [math(\|f\|_{L^p}=\infty)] 또는 [math(\|g\|_{L^q}=\infty)]인 경우 자명하므로 [math(\|f\|_{L^p}, \|g\|_{L^q}<\infty)]로 가정한다.

([math(\mu)]가 유한 측도인 경우) [math(\theta=2^{-1})], [math(q=2)]라 하면 [math(p^{-1}+q^{-1}=1)]이다. [math(X)]의 어느 점에서도 [math(0)]이 아닌 두 단순함수 [math(f, g)]와 두 실수
[math(\displaystyle\gamma=\frac{p}{2-p},\quad\delta=\frac{-p}{2-p})]
에 대하여 함수 [math(\nu:\mathcal{M}\to[0, \infty])]를
[math(\displaystyle\nu(E)=\int_E |f|^\gamma|g|^\delta\,d\mu)]
로 정의하면 [math(\nu)]는 [math((X, \mathcal{M}))] 위의 측도이다. 두 실수
[math(\displaystyle\alpha=\frac{1-p}{2-p},\quad\beta=\frac{1}{2-p})]
에 대하여 [math(h=|f|^\alpha|g|^\beta)]라 하면 [math(L^p(X, \nu))] 노름 [math({\|\cdot\|_{\nu_p}})]의 로그 볼록성에 의하여 [math(\|h\|_{\nu_2}^2\le\|h\|_{\nu_p}\|h\|_{\nu_q})]가 성립한다. [math(f=\sum_{k=1}^m a_k1_{E_k}, g=\sum_{k=1}^n b_k 1_{F_k} )]라 하면
[math(\displaystyle fg=\sum_{i=1}^m\sum_{j=1}^na_ib_j1_{E_i\cap F_j},)]
[math(\begin{aligned}\nu(E_i\cap F_j)&=\int_{E_i\cap F_j}|f|^\gamma|g|^\delta d\mu\\
&=|a_i|^\gamma|b_j|^\delta\mu(E_i\cap F_j)\end{aligned})]
이므로
[math(\displaystyle\begin{aligned}
&\|h\|^2_{\nu_2}\\
&=\int\|f\|^{2\alpha}\|g\|^{2\beta}\,d\nu\\
&=\sum_{i=1}^m\sum_{j=1}^n |a_i|^{2\alpha}|b_j|^{2\beta}\nu(E_i\cap F_j)\\
&=\sum_{i=1}^m\sum_{j=1}^n |a_i|^{2\alpha+\gamma}|b_j|^{2\beta+\delta}\mu(E_i\cap F_j)\\
&=\int |f|^{2\alpha+\gamma}|g|^{2\beta+\delta}\,d\mu\\
&=\int |f||g|\,d\mu\\
&=\|fg\|_{L^1}
\end{aligned})]
이다. 위와 같은 방법으로
[math(\displaystyle\begin{aligned}
\|h\|_{\nu_p}&=\left(\int|f|^{\alpha p}|g|^{\beta p}\, d\nu\right)^{1/p}\\
&=\left(\int |f|^{\alpha p+\gamma}|g|^{\beta p+\delta}\,d\mu\right)^{1/p}\\
&=\left(\int |f|^p|g|^0\,d\mu\right)^{1/p}\\
&=\|f\|_{L^p}
\end{aligned})]
이고 [math(p=q/(q-1))]에서
[math(\displaystyle\beta=\frac{1-q}{2-q},\quad \delta=\frac{q}{2-q})]
이므로 [math(\|h\|_{\nu_q}=\|g\|_{L^q})]이다. 즉, [math(\|fg\|_{L^1}\le\|f\|_{L^p}\|g\|_{L^q})]가 성립한다. 임의의 [math(f\in L^p(X, \mu), g\in L^q(X,\mu))]에 대하여 [math(E=\operatorname{supp}(f)\cap\operatorname{supp}(g))]라 할 때, 각 항이 [math(E)]의 모든 점에서 [math(0)]이 아니고 절댓값이 증가하며 각각 [math(|f|)]와 [math(|g|)]로 수렴하는 두 단순 함수열을 [math(\{f_n\}, \{g_n\})]이라고 하면 유계 수렴 정리에 의해 [math(\|fg\|_{L^1}\le \|f\|_{L^p}\|g\|_{L^q})]가 성립한다.

([math(\mu)]가 [math(\sigma)]-유한 측도인 경우) [math(\{E_n\})]을 [math(X=\bigcup_{n=1}^\infty E_n)]인 증가 유한측도 집합렬이라 하자. 자연수 [math(n)]과 두 함수 [math(f\in L^p, g\in L^q)]에 대하여 [math(f_n=f|_{E_n}, g_n=g|_{E_n})]에 대하여 [math(\|f_ng_n\|_{L^1}\le\|f_n\|_{L^p}\|g_n\|_{L^q})]이다. [math(n\le n+1)]이면
[math(\begin{aligned}\|f_ng_n\|_{L^1}&\le\|f_{n+1}g_{n+1}\|_{L^1},\\
\|f_n\|_{L^p}&\le\|f_{n+1}\|_{L^p},\\
\|g_n\|_{L^q}&\le\|g_{n+1}\|_{L^q}
\end{aligned})]
이므로 [math(n\to\infty)]일 때
[math(\begin{aligned}\|f_ng_n\|_{L^1}&\to\|fg\|_{L^1},\\
\|f_n\|_{L^p}&\to\|f\|_{L^p},\\
\|g_n\|_{L^q}&\to\|g\|_{L^q}
\end{aligned})]
이고 [math(\|fg\|_{L^1}\le\|f\|_{L^p}\|g\|_{L^q})]가 성립한다.

([math(\mu)]가 임의의 측도인 경우) 두 함수 [math(f\in L^p, g\in L^q)]와 [math(X)]의 [math(\sigma)]-유한 집합족 [math(\mathcal{E})]에 대하여 [math(M=\sup_{E\in\mathcal{E}}\|fg|_E\|_{L^1})]이라 하자. [math(\|fg|_{E_n}\|_{L^1})]가 [math(M)]으로 수렴하도록 하는 [math(\mathcal{E})]의 집합렬 [math(\{E_n\})]에 대하여 [math(E=\left(\bigcup_{n=1}^\infty E_n\right)\cup\operatorname{supp}(fg))]이라 하면 [math(E)]는 [math(\sigma)]-유한 집합이다. 따라서
[math(\displaystyle\begin{aligned}\|fg\|_{L^1}&=\|fg|_E\|_{L^1}\\
&\le \|f|_E\|_{L^p}\|g|_E\|_{L^q}\\
&\le \|f\|_{L^p}\|g\|_{L^q}
\end{aligned})]
이다.
}}}||

4.4. 작용소의 보간

리츠 보간 정리(Riesz interpolation theorem)
주어진 지수 [math(1 \le p_1<p_2 \le \infty,1 \le q_1<q_2 \le \infty)]에 연관된 작용소 [math(T : L^{p_1} + L^{p_2} \rightarrow L^{q_1} + L^{q_2})]가 [math(T(L^{p_1}) \subseteq L^{q_1})], [math(T(L^{p_2}) \subseteq L^{q_2})]을 만족하고 정의역 위에서 유계일 때, 임의의 [math(0<\lambda<1)]과
[math( \displaystyle (p,q)= \left( \frac{\lambda}{p_1} + \frac{1-\lambda}{p_2} \right)^{-1}, \left( \frac{\lambda}{q_1} + \frac{1-\lambda}{q_2} \right)^{-1})]
에 대해서 [math(T)]는 [math(L^{p} \rightarrow L^{q})]로의 잘 정의된 유계 연산자이며, 그 크기는 다음의 식으로 제한된다.
[math( \displaystyle \|T|_{L^p}\| \le \|T|_{L^{p_1}} \|^{\lambda} \|T|_{L^{p_2}} \|^{1-\lambda} )]

5. 적용

편미분방정식을 전공하지 않으면 흔히 보는 것은 [math(L^1, L^2, L^{\infty})] 정도일 것이다.

이 중 [math(L^2)] 공간은 많은 분야에서 특별한 지위를 누리고 있는데, [math(L^p)] 공간 중에서 유일하게 내적을 논할 수 있는 공간이 [math(L^2)]이기 때문이다. 일반적인 작용소를 분석하기 가장 쉬운 공간이 스펙트럼 정리의 위력을 자유자재로 사용할 수 있는 힐베르트 공간이며, 많은 이계 미분작용소[2]는 특정 가중치가 주어진 변형된 [math(L^2)] 공간에서의 에르미트 연산자로 간주될 수 있으므로 이 접근 방식은 상당히 범용적인 방법이다. 꼭 미분방정식이 아니더라도 [math(L^2)]는 푸리에 변환이 연산자로 정의되는 유일한 공간인 등등의 여러 가지 특수성을 지니고 있다. 해석학 전공자가 아니라면 이러한 내용들의 응용 예시 중 제일 유명하고 중요한 경우인 양자역학에서 [math(L^2)] 공간을 제일 자주 보게 될 것이다. 양자역학 자체가 내적(브라켓)과 연산자를 힐베르트 공간[3], 위에서 다루는 내용 위에 쌓아올려진 학문이기 때문.

[math(L^1)]과 [math(L^{\infty})] 공간은 정의 자체가 매우 직관적이기 때문에 제일 먼저 배우기도 하며, 유계성을 다루는 곳에서 외려 [math(L^2)]보다도 훨씬 자연스럽게 등장하는 경우가 많다. 다만 연산자로서의 성질을 다수 희생해야 하는 단점이 있어 의외로 [math(L^2)]보다 다루기 불편한 상황들도 있다. 통계학이나 최적화 이론 등등의 응용수학에선 선형 계획법(linear programming) 형태의 대부분 문제에서 [math(L^1)]이나 [math(L^{\infty})] 노름이 적용되고, 이차형식과 유클리드 노름을 다룬다면 [math(L^2)] 공간이 나오는 경우가 많다. 보통 [math(L^2)] 노름은 미분에 대해 최적화하기가 쉽지만 계산하긴 어렵고, [math(L^1)]이나 [math(L^{\infty})] 노름은 반대의 성질을 가지게 된다.

확률론에서의 적률(모멘트, moment)을 높은 차수까지 요구한다면 그 차수만큼의 [math(L^n)]을 요구할 수 있겠지만, 부득이한 상황이 아니고선 적절한 방법으로 꼬리를 잘라내고 일반적인 확률변수에 대해 확장하는 것이 요구되기 때문에 [math(L^2)]를 넘어서는 공간이 잘 나오지는 않는 편이다.

푸리에 변환은 [math(1 \le p \le 2)]이면 [math(L^p \rightarrow L^q)] 위에서 잘 정의되며, 특별히 [math(L^2 \rightarrow L^2)] 범위에서는 동형사상(isomorphism)이 된다.

해석학 내에선 [math(L^p)] 공간은 비교적 단순한 편으로, 더 다양한 상황을 묘사하기 위해선 도함수 등등에 조금 더 다양한 조건이 붙어 있는 여러 가지 함수공간을 생각하기도 한다. 모든 차수의 도함수가 [math(L^p)]에 있는 소볼레프 공간(Sobolev space)이 대표적인 예이다. 이런 상황에서도 특정 편미분방정식을 전공하지 않는 한 대부분의 사람들이 [math(p=1,2,\infty)]가 아닌 다른 특정한 차수의([math(L^3)], [math(L^4)] 같은) 공간을 볼 일은 별로 없다.


[1] [math(\|f+g\|_{L^2}^2+\|f-g\|_{L^2}^2=\int|f+g|^2+\int|f-g|^2=2\int|f|^2+2\int|g|^2=2\|f\|_{L^2}^2+2\|g\|_{L^2}^2)] [2] 정확히 말하면 타원 작용소(elliptic operator)들 [3] 다만 양자역학의 힐베르트 공간 중에서도 [math(L^2)]공간으로 정의되지 않는 것도 많으므로 주의하자.